上一主题:Reading 54: Efficient Capital Markets - LOS b, (Part 1) ~
下一主题:Reading 54: Efficient Capital Markets - LOS a, (Part 2) ~
返回列表 发帖

Reading 54: Efficient Capital Markets - LOS a, (Part 2) ~

6.Which of the following forms of the EMH assumes that no group of investors has monopolistic access to relevant information?

A)   Weak-form.

B)   Semistrong-form.

C)   Both weak and semistrong form.

D)   Strong-form.


7.The strong-form EMH goes beyond the semistrong-form in that it calls for:

A)   a large number of profit maximizing participants.

B)   information to come to the market on a random basis.

C)   cost free availability of all information, both public and private.

D)   all non-market public information should be incorporated into security prices.


8.Which of the following is NOT an assumption behind the semistrong-form of the EMH?

A)   All information is cost-free and available to everyone at the same time.

B)   A large number of profit-maximizing participants.

C)   The timing of news announcements are independent of each other.

D)   Investors adjust their expectations rapidly when confronted with new information.


9.Which of the following statements is INCORRECT?

A)   The strong-form EMH assumes cost free availability of all information, both public and private.

B)   The weak-form EMH suggests that fundamental analysis will not provide excess returns while the semi-strong form suggests that technical analysis cannot achieve excess returns.

C)   The semi-strong form EMH addresses market and non-market public information.

D)   The weak-form EMH states that stock prices reflect historical public market information.


10.The semi-strong form of efficient market hypothesis (EMH) asserts that:

A)   past and future prices exhibit little or no relationship to another.

B)   insider information is incorporated into security prices as it becomes available.

C)   both public and private information is already incorporated into security prices.

D)   all public information is already reflected in security prices.

答案和详解如下:

6.Which of the following forms of the EMH assumes that no group of investors has monopolistic access to relevant information?

A)   Weak-form.

B)   Semistrong-form.

C)   Both weak and semistrong form.

D)   Strong-form.

The correct answer was D)

The strong-form EMH assumes that stock prices fully reflect all information from public and private sources. In addition, no group of investors has monopolistic access to information relevant to the formation of prices.


7.The strong-form EMH goes beyond the semistrong-form in that it calls for:

A)   a large number of profit maximizing participants.

B)   information to come to the market on a random basis.

C)   cost free availability of all information, both public and private.

D)   all non-market public information should be incorporated into security prices.

The correct answer was C)

The strong-form assumes perfect markets in which all information, both public and private, is cost free and available to everyone at the same time.


8.Which of the following is NOT an assumption behind the semistrong-form of the EMH?

A)   All information is cost-free and available to everyone at the same time.

B)   A large number of profit-maximizing participants.

C)   The timing of news announcements are independent of each other.

D)   Investors adjust their expectations rapidly when confronted with new information.

The correct answer was A)

The strong-form EMH assumes all information, both public and private, is cost-free and available to all investors at the same time.


9.Which of the following statements is INCORRECT?

A)   The strong-form EMH assumes cost free availability of all information, both public and private.

B)   The weak-form EMH suggests that fundamental analysis will not provide excess returns while the semi-strong form suggests that technical analysis cannot achieve excess returns.

C)   The semi-strong form EMH addresses market and non-market public information.

D)   The weak-form EMH states that stock prices reflect historical public market information.

The correct answer was B)

The weak-form EMH suggests that technical analysis will not provide excess returns while the semi-strong form suggests that fundamental analysis cannot achieve excess returns. The weak-form EMH assumes the price of a security reflects all currently available historical information. Thus, the past price and volume of trading has no relationship with the future, hence technical analysis is not useful in achieving superior returns.

The other choices are correct. The strong-form EMH states that stock prices reflect all types of information: market, non-public market, and private. No group has monopolistic access to relevant information; thus no group can achieve excess returns. For these assumptions to hold, the strong-form assumes perfect markets – information is free and available to all.


10.The semi-strong form of efficient market hypothesis (EMH) asserts that:

A)   past and future prices exhibit little or no relationship to another.

B)   insider information is incorporated into security prices as it becomes available.

C)   both public and private information is already incorporated into security prices.

D)   all public information is already reflected in security prices.

The correct answer was D)

Semi-strong EMH states that publicly available information cannot be used to consistently beat the market performance.

TOP

返回列表
上一主题:Reading 54: Efficient Capital Markets - LOS b, (Part 1) ~
下一主题:Reading 54: Efficient Capital Markets - LOS a, (Part 2) ~